Write in expression
multiply the quotient of 10 and r by 4

Answers

Answer 1

Answer:

10/r x 4

Step-by-step explanation:


Related Questions

The formula for the circumference of a circle is C = 2r, where r is the radius and C is the circumference. The equation solved for r is r = r equals StartFraction C Over 2 pi EndFraction..

Find the radius of a circle that has a circumference of 16.

r = 4
r = 8
r = 12
r = 16

Answers

Answer:

r=8

Step-by-step explanation:

Because 16 divided by 2 is 8

The radius of a circle has a circumference of 16 at r = 8.

What is the radius of a circle?

Given: the formula for the circumference of a circle is C = 2r, where r is the radius and C is the circumference.

The circumference of a circle is equal to π.

π ≈ 3.14

multiplied by the diameter of the circle.

Therefore, the diameter has twice the length of the radius.

In equation form:

2r = d.

2r = 16.

r = 8.

Note, 2r = d,

Then, the equation C = 2πr holds and can be used in place of C = πd.

The radius of a circle has a circumference of 16 at r = 8.

Learn more about the radius of a circle

https://brainly.com/question/22515423

#SPJ2

1.56666666667 as a fraction? pls help asap

Answers

to do this all you have to do is make the denominator 1 and the numerator should be 1.566666667

.please answer this question i will mark it brainliest

Answers

Answer:

a = 20.79cm ( 2 decimal places)

Step-by-step explanation:

find the length of the identical sides (i name it "x") first using Pythagoras theorem

[tex]14.8^{2}[/tex] = [tex]x^{2} + x^{2}[/tex]

219.04 = [tex]2x^{2}[/tex]

[tex]x^{2}[/tex] = 219.04/2 = 109.52

x = [tex]\sqrt{109.52}[/tex]

now find a,

[tex]a^{2}[/tex] = [tex](\sqrt{109.52}) ^{2}[/tex] + [tex](7.5 + \sqrt{109.52}) ^{2}[/tex]

[tex]a^{2}[/tex] = 109.52 + 322.7477 = 432.2677

a = [tex]\sqrt{432.2677}[/tex] = 20.791048

Somone I need help with this? Number 7

Answers

Answer:

Shape

Step-by-step explanation:

Shape because if you make the boat the right shape, it has a better chance of floating.

Its definitely shape!

Patrick and his brother each start a savings account. Patrick begins with $200
and plans to deposit an additional $25 per month. His brother begins with
$150 and plans to deposit an additional $35 per month. After how many
months will the two brothers have the same amount in their accounts?

Answers

Answer:

Both brothers will have the same amount of money is their saving accounts after six months.

Step-by-step explanation:

I worked this out by finding out the first 8 months of savings for both brothers:

Patrick:               His brother:

200                     150

225                     185

250                     220

275                     253

300                     290

325                     325

375                     360

400                     395

I then looked across my results to see that at 6 months both brothers had $325.

Hope this helped.

Kathleen and Arnob both run from the park entrance along a loop. Kathleen starts walking from the park entrance and gets a 5-mile head start on Arnob. The graph shows how far they have both traveled. A coordinate grid labeled Distances Traveled with Time in minutes along the horizontal x-axis and Distance in miles along the vertical y-axis with 2 lines. The first line is labeled y equals (StartFraction 1 over 15 EndFraction) x plus 5 and passes through (0, 5) and (60, 9). The second line is labeled y equals (StartFraction 2 over 15 EndFraction) x. The lines appear to intersect at between 70 and 80 along the x-axis and 10 on the y-axis. How many minutes does it take Arnob to catch up to Kathleen? 9.8 10 73.5 75

Answers

Answer:

75

Step-by-step explanation:

It's D on edg

The number of minutes it take Arnob to catch up to Kathleen is 75 minutes.

Number of minutes

Given:

Mile head start=5 mile

Distance along y-axis=2 lines

Hence:

y=1/15x+5

=y=2/15x

So,

1/15x+5=2/15x

1/15x=5

x=75 minutes

The correct option is D.

Learn more about number of minutes here:https://brainly.com/question/411661

#SPJ9


If p | q, which of the following statements is true for the given figure?

Answers

Answer:

The answer is D.

Step-by-step explanation:

Angle 1 and 2 are equivalent to the measure of angles 3+4. We know this because both sides on the middle line form a 90 degree angle

I hope this helps :)

Two sides of an obtuse triangle measure 10 inches and 15 inches. The length of longest side is unknown. What is the smallest possible whole-number length of the unknown side? 18 inches 19 inches 24 inches 25 inches

Answers

Answer:

19 inches is the correct answer.

Step-by-step explanation:

Side 1 = 10 inches

Side 2 = 15 inches

It is given that it is an obtuse angle triangle i.e. the largest angle of the triangle is more than [tex]90^\circ[/tex].

Let us suppose, that the triangle be a right angle triangle, then the largest angle will be [tex]90^\circ[/tex]. The largest side is known as Hypotenuse.

As per pythagoras theorem:

[tex]Hypotenuse^{2} = Base^{2} + Height^{2}[/tex]

[tex]\Rightarrow 10^{2} + 15^{2} = Hypotenuse^2\\\Rightarrow 325 = Hypotenuse^2\\\Rightarrow Hypotenuse = 18.04\ inches[/tex]

So, largest side comes out be 18.04 inches.

But actually, the largest angle is more than [tex]90^\circ[/tex] so the largest side will also be greater than 18.04 inches.

Hence, the answer is:

Smallest possible whole number length of unknown side will be 19 inches.

Answer:

I did the test and thecorrect option was 18.

What is the area of a parallelogram when the height is 10 and the base is 15

Answers

Answer:

150

Step-by-step explanation:

A=bh

A=15×10

A=150

Answer:

150

Step-by-step explanation:

it is just like finding the area of a rectangle! 10*15=150

Please help! Correct answer only please! I need to finish this assignment by today.

On a residential single lane road there was a wreck that backed up traffic for 4 miles. 70% of the traffic consists of cars and 30% of the traffic consists of trucks. the average distance between vehicles is 3 feet. the average length of a car is 13.5 feet and the average length of a truck is 20 feet. estimate how many vehicles are stuck in the traffic jam.

(hint: there are 5280 feet in 1 mile)


A) 276

B) 896

C) 1172

D) 1412

Answers

Answer: 1412 vehicles.

Step-by-step explanation: I calculated my answers and I approximately got this anwse so... Hope it helps.

What is the value of x for the point with a y-value of 6?

Answers

4 is the correct answer or x=4 ✌️
The answer would be 4.
If you look at the graph and look for the number six along the y-axis (y), you can find that the point is above the number 4 on the x-axis (x).

If ABCD is dilated by a factor of , the
coordinate of D' would be:

PLEASE HELP I WILL GIVE BRAINLIEST!!!!!!!!

Answers

Answer:

(2,-1)

Step-by-step explanation:

Multiply the original coordinates by 1/3

Answer will be (2,-1)

Using the greatest common factor, reduce the fraction \frac{32}{48} ​48 ​ ​32 ​​ to its simplest form

Answers

Answer:

2/3

Step-by-step explanation:

32/48= 16*2/16*3= 2/3

A figure is a regular n-gon. Which of the following
expressions represents the sum of the measures of
the exterior angles of the polygon, one at each
vertex?
A. (180 - 10°
360
B. 180
12
C. (n - 2)180°
D. 360°

Answers

Answer:

The correct answer is D. 360°

Step-by-step explanation:

The sum of all the exterior angles of a regular n-gon or a polygon is always equal to 360°.

Let us consider the image attached.

First of all let us consider the triangle.

As per the property of exterior angles: The exterior angle is equal to the sum of 2 opposite interior angles.

[tex]\Rightarrow \angle 1=\angle B + \angle C\\\Rightarrow \angle 2=\angle A + \angle C\\\Rightarrow \angle 3=\angle A + \angle B[/tex]

Where [tex]\angle A, \angle B\ and\ \angle C[/tex] are the interior angles of the triangle.

Adding the above equations:

[tex]\angle 1 +\angle 2 +\angle 3 = 2 \times (\angle A +\angle B +\angle C )[/tex]

The sum of interior angles of a triangle is always equal to 180°

[tex]\angle A +\angle B +\angle C = 180^\circ[/tex]

[tex]\angle 1 +\angle 2 +\angle 3 = 2 \times 180^\circ\\\Rightarrow \angle 1 +\angle 2 +\angle 3 = 360^\circ[/tex]

Hence, proved that the sum of exterior angles is [tex]360^\circ[/tex].

So, the correct answer is D. [tex]360^\circ[/tex]

the simple interest on $400 at 5% per annum for 2 years is?
a) 400*5*2/100
b) 400*5/2*100
c) 400*2/5*100
d) 400*100/2*5​

Answers

Answer:

[tex] \frac{400 \times 5 \times 2}{100} [/tex]

Answer A is correct

Step-by-step explanation:

[tex]p = 400 \\ r = 5\% \\ t = 2 \: \: years[/tex]

Now let's find the simple interest.

[tex] \frac{prt}{100 } \\ = \frac{40 0 \times 5 \times 2}{100} \\ = 40[/tex]

hope this helps

brainliest appreciated

good luck! have a nice day!

Answer:

It's a).

Step-by-step explanation:

I = PRT/100

- where I = the interest , P = initial amount, R = annual rate and T is the time in years.

So the answer is (400*5*2) / 100.

find f(-3) if f(x) = x^2

Answers

Answer:

=

1

2

×

3

2

=

1

×

9

=

9

Step-by-step explanation:

f

(

3

)

=

9

Explanation:

f

(

x

)

=

x

2

To find f(-3) replace

x

in the function by

3

Therefore:

f

(

3

)

=

(

3

)

2

=

1

2

×

3

2

=

1

×

9

=

9

Answer:

9

Step-by-step explanation:

f(x) = x²

f(-3) =(-3)² = (-3)*(-3) = 9

Will give brainliest answer What is the value of a21? A.36 B. 58 C.60 D. 41

Answers

Answer:

41

Step-by-step explanation:

Matrix Multiplication follows a row-column format. In order to compute this, you must be familiar with vector dot products.

With that in mind, lets get straight into it.

The order that matrix multiplication follows means that the terms in the result are filled in left to right, then top to bottom.

Therefore, a21 will be the 3 value that is computed. This is important becuase this allows to directly compute a21, instead of using up a lot of time computing all the values before.

As a21 is located in the bottom row 1st column, we take the dot product of the 2nd row in matrix 1 and the 1 column in matrix two.

So we have:

(7 2) dot (5 3) = 7*5 + 2*3 = 41

Jana has a number cube numbered 1 to 6. She plans to roll the cube 78 times. Predict the number of times Jana will roll a 2

Answers

Answer:

13 times

Step-by-step explanation:

78 divided by six

(could be wrong)

Scenario: Mike is buying a large pizza. The pizza
costs $8.00 plus $0.50 for each additional topping.
Identify the y-intercept and slope.
y-intercept:
Slope:

Answers

The y int is 8 and the slope is 1/2

Lisa has a gym membership at Lifetime Fitness. It costs $29.95 to join and she also pays a monthly charge. Lisa has been a member for 11 months. She has paid a total of $298.90 for her membership. Write an equation that could be used to find, m, the monthly membership fee.
What is the monthly membership fee?

Answers

Answer:

298.90 - 29.95 = (ANSWER) divide it by 11 then you will have the monthly cost

Step-by-step explanation:

Answer:

monthly paid= 24.45

Step-by-step explanation:

if we take membership join fee=x=29.95

and the monthly fee=m

so the equation will be

11 m + x =298.90

so by substitute x

m=24.45 $/month

I’m not very good at this kind of stuff.

Answers

I’m not too sure but I know you might have to cube it again instead of just multiply the two

For f(x) = 4x+1 and g(x) = x^2-5 find (f+g)(x)

Answers

Answer:

Step-by-step explanation:

(f+g)(x) = f(x) + g(x)

           = 4x + 1 + x² - 5

            = x² + 4x  - 4

Answer:

x^2 +4x -4

Step-by-step explanation:

f(x) = 4x+1

g(x) = x^2-5

(f+g)(x) =  4x+1 + x^2 -5

Combine like terms

            = x^2 +4x +1-5

              =x^2 +4x -4

If A = 3x² + 5x - 6 and B --2x2 - 6x + 7, then A - B equals?

Answers

Answer:

x^2 + 11x - 13

Step-by-step explanation:

A = 3x^2 + 5x - 6

B = 2x^2 - 6x + 7

=> A - B = (3x^2 + 5x - 6) - (2x^2 - 6x + 7)

             = (3x^2 - 2x^2) + (5x + 6x) - (6 + 7)

             = x^2 + 11x - 13

I hope this helps!

the first and third terms in the following Fibonacci sequence are x and y.
Write down algebraic expressions for the second, fourth and fifths terms.
plz reply ASAP

Answers

Answer:

The second term of the sequence is y-xThe fourth term of the sequence is 2y-xThe fifth term of the sequence is 3y-x

Step-by-step explanation:

The next term of a Fibonacci sequence is generated by taking the sum of the 2 preceding value of the sequence. Given the the first and third term of a Fibonacci sequence to be x and y

x, _, y...

let the second term be a, fourth term be b and fifth term be c to have the sequence;

x, a, y, b, c...

According to the definition;

x+a = y... 1

a+y = b... 2 and;

y+b = c... 3

From equation 1, a = y-x

Substituting a = y-x into equation 2 to get b we have;

(y-x)+y = b

2y-x = b

b = 2y-x

Substituting b = 2y-x into equation 3 to get 'c'

y +  2y-x = c

3y-x = c

c = 3y-x

The second term of the sequence is y-x

The fourth term of the sequence is 2y-x

The fifth term of the sequence is 3y-x

find the value of x part 2​

Answers

Answer:

x= -10

Step-by-step explanation:

Please see attached picture for full solution.


Solve for h.
h–2–17≤–1

Answers

Answer:

h≤18

Step-by-step explanation:

Step  1  :

Solve Basic Inequality : 1.1      

Add  18  to both sides h ≤ 18

Inequality Plot

Inequality plot for

        h  - 18.000  ≤  0

Answer: h≤18

Hope this helps.

Find the product.

(n3)2 · (n5)4

Answers

Answer:

[tex]n^{26}[/tex]

Step-by-step explanation:

[tex]n^{3 \times 2} \times n^{5 \times 4}[/tex]

[tex]n^{6} \times n^{20}[/tex]

[tex]n^{6+20}[/tex]

[tex]n^{26}[/tex]

Answer:

N^26

Step-by-step explanation:

N ^3^2 = n^6

N^5^4 = n^20

Multiply. Use laws of exponents

N^6 x n^20 = n^20+6 = n^26

Hope this helps

Find the surface area of the composite figure. Round the answer to the nearest tenth.
3 cm
4 cm
5 cm
11 cm
The surface area of the figure is
cm

Answers

The answer is 4 cm I think idk

What are the slope and y-intercept of the line represented by
the points shown in the table?

Answers

Answer:

To find the y-intercept, graph the points, and draw a line that passes through them. the place that it passes the y-axis is the y-intercept. To find the slope, find two points on the line. Take the increase in the y-coordinate and divide it by the increase in the x-coordinate.

Step-by-step explanation:

Answer:

B

Step-by-step explanation:

Please solve idk how to

Answers

Answer:

You have the right answer.

Step-by-step explanation:

The image might be blurry but its the best I can post.

That's solving it without the x and y values. Its still the answer you picked.

Hope this helps, have a good day! :)

(Brainliest would be appreciated?)

Answer:

5√xy

Step-by-step explanation:

√25x²y²/√xy= √25x²y²/xy= √25xy= 5√xy

Other Questions
Why is it important to talk with friends and loved ones during a crisis related to the coronavirus??? Andrs, today Paulina is in the market to buy fruit, but she does not have a car. Before ________ Paulina from the market, you have to ________. There is no gasoline. A) give back; deposit the check B)to write; change the tire C)collect; fill the tank D) come; close the trunk A market maker faces the following demand and supply for widgets. Eleven buyers are willing to buy at the following prices: $15, $14, $13, $12, $11, $10, $9, $8, $7, $6, $5. Eleven sellers are also willing to sell at the same prices. If the market maker is free to choose the number of transactions he can make, what is his maximum profit what are the properties of sound waves 10 grams of sodium hydroxide, NaOH, is dissolved in 0.25 liters of solution. Determine the molarity (M) Select the word from the drop-down menu that most accurately completes the sentence A ______ text is one examples how to do something or how a process works. Some examples of these informational texts include recipes, how-to manuals, and legal documents A. fictional B. historical C. persuasiveD. technical does anyone know the answer??? What is the formula for this ionic compound and tha name ? A book sold 32100 copies in its first month of release. Suppose this represents 6.2% of the number of copies sold to date. How many copies have sold to date. What is the additive inverse of the complex number -3 + 3/?-8 - 3-8 + 3;O 8 - 31O 8 + 3 what do you think 4040 isAnd tell how you got your answer For the just completed year, Hanna Company had net income of $41,500. Balances in the companys current asset and current liability accounts at the beginning and end of the year were as follows: December 31 End of Year Beginning of Year Current assets: Cash and cash equivalents $ 57,000 $ 78,000 Accounts receivable $ 162,000 $ 188,000 Inventory $ 442,000 $ 370,000 Prepaid expenses $ 11,500 $ 14,000 Current liabilities: Accounts payable $ 370,000 $ 384,000 Accrued liabilities $ 8,500 $ 12,000 Income taxes payable $ 36,000 $ 27,000 The Accumulated Depreciation account had total credits of $48,000 during the year. Hanna Company did not record any gains or losses during the year.Required: Use the indirect method to determine the net cash provided by (or used in) operating activities for the year. Please help me with this its 1 problem should not be hard for most just that I'm tired and would need some help if somebody can help me! Read and choose the option that best answers the question. Hi, I'm Maarit, and welcome to Xcanatun, my favorite city in Mexico. For a small town, Xcanatun has an amazing history that goes back thousands of years. The walls of my house are made from stones that were found in the area. They were once used by an ancient Maya tribe to build their beautiful temples and carvings. Through these stones I feel connected to my remarkable ancestors. My family tends animals and farms the land. We grow sweet potatoes, corn, beans, chilies, and squash, just as our Maya ancestors have been doing for thousands of years. Based on the text, what Maya tradition has continued in Maarit's household? A. The house B. The language C. The job D. The songs Cul es el tiempo del verbo?Esta tarde almuerzo con mi familia.a. presenteb. pasadoc. futuro Read the following sentence.If a person wants to succeed in life, you really have to work at it.If the pronoun and antecedent in the sentence above are correct as is, write "correct" and explain the reason the pronoun choice is correct. If the sentence contains a pronoun error, rewrite the sentence to correct the error, then explain the reason the pronoun used was an error. Joe makes an hourly salary of $8.10 for 40 regular hours of work. For each hour worked over 40 hours, he is paid at a rate of $12.15 per hour. Last week Joe worked 45 hours. He has withholdings of social security tax (6.2%), medicare tax (1.45%), federal tax ($61.12), state tax ($21.03), city tax ($6.01), pension ($4.12), disability insurance ($1.31), medical insurance ($13.05), and dental insurance ($5.46).a. What is Joes Medicare tax deduction?b. What is Joes Social Security deduction? c. What is Joes Medicare tax deduction? d. Joes other deductions are: federal tax 21.03, city tax 4.12, disability insurance 13.05, and dental insurance $5.46. What are Joes total deductions for this pay period? e. What is Joes net pay for this pay period?f. If Joe pays the same amount for medical insurance each weekly pay period, what is his annual premium? g. Does Joe receive time-and-a-half, double-time, or triple-time for each hour of overtime? What historical development does the cartoon depict?"Yes, You Remembered MeTEORISource: C. D. Batchelor, New York Daily News, October 11, 1936President Franklin Roosevelt's court packing scheme.President Franklin Roosevelt's New Deal programs.President Franklin Roosevelt's bargain with the Republican party.President Franklin Roosevelt's commitment to traditional American values. An objects weight on Earth varies directly to its weight on the Moon. If a man weighs 180 kg on Earth, then he will weigh 30 kg on the Moon. Set up an algebraic equation that expresses the weight on Earth in terms of the weight on the Moon and use it to determine the weight of a woman on the Moon if she weighs 120 kg on Earth. CHOICES:Which three human activities lead to a loss of terrestrial habitat?-conservation-farming-deforestation-construction